- PowerScore Staff
- Posts: 5972
- Joined: Mar 25, 2011
- Fri Mar 09, 2012 12:37 pm
#85108
Complete Question Explanation
(The complete setup for this game can be found here: lsat/viewtopic.php?f=351&t=3899)
The correct answer choice is (A)
The condition in this question stem stipulates that no color is used twice, and thus each light must be a different color. From a direct application of the rules, this automatically affects the second and third rules:
However, the other two incorrect answer choices can also be eliminated by examining the third rule more closely:
(The complete setup for this game can be found here: lsat/viewtopic.php?f=351&t=3899)
The correct answer choice is (A)
The condition in this question stem stipulates that no color is used twice, and thus each light must be a different color. From a direct application of the rules, this automatically affects the second and third rules:
Because the second rule stipulates that when light 2 is green then light 1 must also be green, which would violate the condition in this question stem, we can determine that light 2 can never be green. This immediately eliminates answer choice (D) from consideration.
In the third rule, when light 3 is purple, then light 2 must also be purple. Accordingly, in this question light 3 can never be purple, and this information can be used to eliminate answer choice (E).
However, the other two incorrect answer choices can also be eliminated by examining the third rule more closely:
In the third rule, when light 3 is yellow, then light 2 must be purple. Answer choice (C), which states that light 1 is purple and light 3 is yellow, would thus cause purple to be used twice (in lights 1 and 2), a violation of the condition in the question stem. Thus, answer choice (C) cannot be true and is incorrect.
From the contrapositive of the third rule, when light 2 is yellow or purple, then light 3 must be green. Answer choice (B), which states that light 1 is green and light 2 is yellow, would thus cause green to be used twice (in lights 1 and 3), a violation of the condition in the question stem. Thus, answer choice (B) cannot be true and is incorrect.Therefore, answer choice (A) is the correct answer, and is possible under the following solution: G-P-Y.
Dave Killoran
PowerScore Test Preparation
Follow me on X/Twitter at http://twitter.com/DaveKilloran
My LSAT Articles: http://blog.powerscore.com/lsat/author/dave-killoran
PowerScore Podcast: http://www.powerscore.com/lsat/podcast/
PowerScore Test Preparation
Follow me on X/Twitter at http://twitter.com/DaveKilloran
My LSAT Articles: http://blog.powerscore.com/lsat/author/dave-killoran
PowerScore Podcast: http://www.powerscore.com/lsat/podcast/